Inequalities trick

This topic has expert replies
Senior | Next Rank: 100 Posts
Posts: 63
Joined: Mon Jul 20, 2009 11:48 am
GMAT Score:730

Inequalities trick

by rainmaker » Wed Apr 07, 2010 4:33 pm
If -5 <= x <= 3, then of the following is correct?

(A) |x| <= 3
(B) |x| <= 5
(C) |x - 2| <= 3
(D) |x - 1| <= 4
(E) |x +1| <= 4

Please explain how you arrived at the answer and if possible a strategy/trick to solve such problems.

Thanks

Master | Next Rank: 500 Posts
Posts: 268
Joined: Wed Mar 17, 2010 2:32 am
Thanked: 17 times

by this_time_i_will » Wed Apr 07, 2010 4:54 pm
Just remember two simple points for inequalities:

I. if |x| <= b & b>0 then -b <= x <= b...that is x lies between -b & b, iclusive
II. if |x| >= b & b>0 , then either x <= -b or x >= b...that is x lies outside the range -b to b, exclusive.

in the backdrop of above information, lets attack answer choices.

given: -5 <= x <= 3

A. -3 <= x <= 3....fits the bill, inside given range

B. -5 <=x <= 5....wrong option, some positive values outside range

C. -3 <= x-2 <= 3 => -1 <= x <= 5....wrong option

D. -3 <= x <= 5 ....wrong option

E.-5 <= x <= 3...better than A.
Last edited by this_time_i_will on Wed Apr 07, 2010 7:17 pm, edited 1 time in total.

User avatar
Legendary Member
Posts: 1560
Joined: Tue Nov 17, 2009 2:38 am
Thanked: 137 times
Followed by:5 members

by thephoenix » Wed Apr 07, 2010 7:00 pm
imo e
its the rephrase of q

User avatar
Master | Next Rank: 500 Posts
Posts: 435
Joined: Mon Mar 15, 2010 6:15 am
Thanked: 32 times
Followed by:1 members

by eaakbari » Wed Apr 07, 2010 10:02 pm
Options


A - If x = -5 then it does not satisfy ;Incorrect
B - the given inequality is a subset of this inequality, Hence all values satisfy. Maybe Correct
C - iF X = -4 . Incorrect
D - If x = -5 . Incorrect
E - All values of x satisfy. Maybe Correct


I cant see how which one is more correct then the other from B and E. Do help

Legendary Member
Posts: 610
Joined: Fri Jan 15, 2010 12:33 am
Thanked: 47 times
Followed by:2 members

by kstv » Wed Apr 07, 2010 11:48 pm
rainmaker wrote:If -5 <= x <= 3, then of the following is correct? (A) |x| <= 3
(B) |x| <= 5 (C) |x - 2| <= 3 (D) |x - 1| <= 4 (E) |x +1| <= 4
Please explain how you arrived at the answer and if possible a strategy/trick to solve such problems.
Thanks
Using a number line and plugin the extreme values possible under each option.
-5 <= x <= 3 so the number line will be

-5 ]-------------------------------0--------------------------[3
x is any value between -5 and +3
Pick values to prove/disprove each option
A. if x = -4 |x| > 3 Eliminate B. if x = -5 |x| < = 5 but if x = 3 |x| < = 5
C. if x = -4 |x -2| > 6 Eliminate D. if x = -5 |x-1| > 6 Eliminate
E. if x = 3 |x+1| <= 4 but if x = -5 |x+1| is still <= 4
notice B and E are the same expression.

I am editing/ highlighting it as from the Expert reply it is obvious that this is an error. B and E are not the same expression.
Last edited by kstv on Thu Apr 08, 2010 9:22 am, edited 1 time in total.

Newbie | Next Rank: 10 Posts
Posts: 2
Joined: Thu Apr 08, 2010 12:03 am

by phuonghang44 » Thu Apr 08, 2010 12:21 am
rainmaker wrote:If -5 <= x <= 3, then of the following is correct?

(A) |x| <= 3
(B) |x| <= 5
(C) |x - 2| <= 3
(D) |x - 1| <= 4
(E) |x +1| <= 4

Please explain how you arrived at the answer and if possible a strategy/trick to solve such problems.

Thanks
Hi,

I think E will be correct.
|x +1| <=4
<=> x+1<=4 and x+1=> -4 (mathematic formula)
<=> x<= 3 and x=>-5
Hope that it helps.

User avatar
Master | Next Rank: 500 Posts
Posts: 435
Joined: Mon Mar 15, 2010 6:15 am
Thanked: 32 times
Followed by:1 members

by eaakbari » Thu Apr 08, 2010 3:06 am
@ kstv
So what is your final answer?
@ phuonghang44
Why not B

@ any1 else reading this
How do you cut down from options B and E
Whether you think you can or can't, you're right.
- Henry Ford

User avatar
GMAT Instructor
Posts: 22
Joined: Mon Mar 15, 2010 2:42 pm
Location: Brooklyn, NY
Thanked: 14 times
Followed by:4 members

by stephen@knewton » Thu Apr 08, 2010 5:01 am
Hey guys,

A couple of strategy points on this one:

- Sometimes in dealing with inequalities, it's helpful to consider the boundaries. You can do this by considering only the "or equal to" portion of the inequality (or, if it's not an inclusive boundary, just pretend it is!). As long as you are careful about which regions make the inequality true (in this case, inside the boundaries), this technique can make things very quick and easy.

STEP 1: The boundaries of the prompt are given to us fairly straighforwardly: they are -5 and 3.

STEP 2: Let's have a look at the answer choices. We'll try to figure out what values of X would make a similar EQUALITY true:

(In case anyone needs a refresher, absolute value means the distance from zero of what's inside the bars. In other words, negatives become positive, positives stay positive).

(A) |x| <= 3 ... the values of X that make this true at the boundaries would be -3 and 3.
(B) |x| <= 5 ... likewise -5 and 5
(C) |x - 2| <= 3 ... -1 and 5
(D) |x - 1| <= 4 ... -3 and 5
(E) |x +1| <= 4 ... -5 and 3

STEP 3: As the first response to this thread noted, some of these answer choices include regions that are "not allowed" by the prompt. These are answer choices B, C and D, and they can be eliminated. By looking at the boundaries in this way, you'll also notice that choices (B) and (E) are actually not describing the same values of X. Remember that we're not allowed to add or subtract to both sides of an inequality from within a absolute value!

STEP 4: Choosing between (A) and (E), both of which describe regions which make the prompt true, is the hardest part. We need to know what the GMAT expects of us. In questions worded as this one is, they are looking for another equation that COMPLETELY and ACCURATELY describes the allowable values for X. Answer choice (A) is wrong because it is incomplete; it fails to describe the region between -5 and -3 that satisfies the prompt.

One last cool absolute value trick, for fun: the form of inequality presented in (C), (D) and (E) describe something very useful. Generically: |x-a|=b describes the two values that are B away from A. In other words, C is all the numbers less than or equal to 3 away from 2. D is all the numbers less than or equal to 4 away from 1. And E is all the numbers less than or equal to 4 away from -1 ... our answer! When you see that form of inequality on the GMAT, this can a great shortcut ... just don't get it backwards!!

Hope this helps a little ...

Cheers, Steve P.
Stephen
GMAT Instructor
Knewton Inc.

User avatar
GMAT Instructor
Posts: 22
Joined: Mon Mar 15, 2010 2:42 pm
Location: Brooklyn, NY
Thanked: 14 times
Followed by:4 members

by stephen@knewton » Thu Apr 08, 2010 5:03 am
... and since I also teach sentence correction, I should correct my last paragraph. The form DESCRIBES.

:)
Stephen
GMAT Instructor
Knewton Inc.

User avatar
Legendary Member
Posts: 526
Joined: Sat Feb 21, 2009 11:47 pm
Location: India
Thanked: 68 times
GMAT Score:680

by harshavardhanc » Thu Apr 08, 2010 6:16 am
stephen@knewton wrote:
One last cool absolute value trick, for fun: the form of inequality presented in (C), (D) and (E) describe something very useful. Generically: |x-a|=b describes the two values that are B away from A. In other words, C is all the numbers less than or equal to 3 away from 2. D is all the numbers less than or equal to 4 away from 1. And E is all the numbers less than or equal to 4 away from -1 ... our answer! When you see that form of inequality on the GMAT, this can a great shortcut ... just don't get it backwards!!

Soooooooooper!!!!!
Regards,
Harsha

Newbie | Next Rank: 10 Posts
Posts: 2
Joined: Thu Apr 08, 2010 12:03 am

by phuonghang44 » Thu Apr 08, 2010 8:44 am
eaakbari wrote:@ kstv
So what is your final answer?
@ phuonghang44
Why not B

@ any1 else reading this
How do you cut down from options B and E
Hallo eaakbari,

They give you a calculation which is A,B,C,D,E choice. Your task is to solve it to find out the x value. After u find out, then check it with the criteria which is the value of x given in the theory above which is -5<=x<=3. E is the most correct therefore E is the best choice.

Also I would like to mention the formula:
1. |x|=a
<=> x=+/-a
2. |x|<=a
<=> x<=a AND x>= - a (it is And not Or).

You apply no.2 in solving A, B, C, D, E then you will find out what I mean. For such calculation, when you get familiar, you will easily do mental calculation and can come up with the answer in just a few second.

hope that it helps.

Also I saw a very useful post after your post. You can refer for further info.

Senior | Next Rank: 100 Posts
Posts: 63
Joined: Mon Jul 20, 2009 11:48 am
GMAT Score:730

by rainmaker » Thu Apr 08, 2010 3:57 pm
Thanks guys for the great tips!